The local library lends an average of 68.24 books to readers each day. What is the average number of books lent to readers in a week?

Answers

Answer 1

Answer:

Step-by-step explanation:

Set this up as a proportion and solve. The ratios are books:day, so keep in mind that a week is 7 days because your values have to be the same

[tex]\frac{books}{day}:\frac{68.24}{1}=\frac{x}{7}[/tex] and cross multiply to solve for x, the number of books lent out in a week:

x = 68.24(7) so

x = 477.68 books in a week


Related Questions

The product of integers a,b,c and d is 546 and if 1<a<b<c<d, what is the value of b+c?​

Answers

Hello,

546=2*3*7*13

a=2, b=3,c=7,d=13

b+c=3+7=10

Cayden has several screws on a scale, and the scale reads 80.955 cayden add 1 more screw and the scale reads 84.81

Answers

Answer:

-3.855

Step-by-step explanation:

Not sure if you need the weight of each screw! but you just subtract 80.955-84.81

Answer:

1

Step-by-step explanation:

itnisbbsndnfnfnfnnfnfnfncjcjccjcj

someone help me for this algebra task please

Answers

i just need to answer questions

the area of a rectangular bathroom mirror is 20 square feet. it is 2 feet tall. how wide is it?

Answers

Area = length x width

Fill in the given information

20 = 2 x width

Solve for width by dividing both sides by 2

Width =20/2

Width = 10 feet

Answer:

It is 10 feet wide

Step-by-step explanation:

The area of a rectangle is:

A = Length x width

So if we have the area, finding the width means diving the area by the given length.

In this case, the area of the rectangle is 20 square feet and the length is 2 feet:

20/2 = 10

Therefore the missing width is 10 feet

Help me?
185/100 + 50% + 4%=?

Answers

Answer:

2.39

Step-by-step explanation:

Convert everything to a decimal.  

To do that with a fraction divide the numerator by the denominator.  

185 divided by 100 = 1.85

185/100 = 1.85

To convert a percent to a decimal divide the number by 100.

50 divided by 100 = 0.5

50/100 = 0.5

4 divided by 100 = 0.04

4/100 = 0.04

Now add:

1.85 + 0.5 + 0.04 = 2.39

Greg buys 60 garden plants at a cost price of $2.00 each to sell in his shop. He sells 25 of them at the profit of 75% and 18 of them at the profit of 35%. He sells the rest of the plants for 4/5 of the cost price calculate the profit or loss he makes from selling 60 plants stating if it is a profit or loss

Answers

Answer:

$43.30 profit

Step-by-step explanation:

Total cost of plant:

60*2 = 120

Greg makes total of:

25*(2 + 0.75*2) + 18*(2 + 0.35*2) + (60 - 25 - 18)*2*4/5 = 163.3

Since Greg mare than cost, he has a profit and the amount is:

163.3 - 120 = 43.3

pls help me don't know what to do

Answers

Answer:

x=15

Step-by-step explanation:

The 60 degree angle and the (x+45) degree angle are both the same degree because they are vertical angles.

So to solve, just subtract 45 from 60

60-45=15

That's your answer!

Hope this helps!

On the unit circle, which of the following angles has the terminal point
coordinates.
A. 45
B. 135
C. 225
D. 315

Answers

Answer: C. 225

Step-by-step explanation:

The formula for the circumference of a circle is R = c/2(pi)

Find the radius of a circle that has a circumference of 16(pi)

A) r = 4
B) r = 8
C) r = 12
D) r = 16

Answers

The answer is B) r= 8

jordan wants to save to buy a car and decides to open a banking account that is offering an interest rate of 4.5% compounded annually how much will jordan have in the account after 5 years it he deposits $7,000 today?

Answers

Answer:

8,723.27$

Step-by-step explanation:

What’s the equation of the line that passes through the point (-4,4) and has a slope of 3/4

Answers

Answer:

y-y1=m(x-x1)

y-4=3/4(x+4)

y=3/4x+7

Select the expression that represents the following statement: add 24 to the quotient of 16 and 8.

Answers

Answer:

16/8 + 24

Step-by-step explanation:

2 1/4 x 3 1/5 brainliest

Answers

Answer:

36/5

Step-by-step explanation:

9/4×16/5

144/20

36/5

hope this is helpful

Answer:

[tex]7\frac{1}{5}[/tex]

Step-by-step explanation:

1. start by turning the fractions improper fractions:

[tex]2\frac{1}{4} =\frac{9}{4}[/tex]

[tex]3\frac{1}{5} =\frac{16}{5}[/tex]

2. then multiply them together:

[tex]\frac{9}{4}[/tex] x [tex]\frac{16}{5}[/tex] = [tex]\frac{144}{20}[/tex]

3. then simplify the fraction:

[tex]\frac{144}{20}[/tex][tex]=\frac{36}{5}[/tex]

4. turn it into a proper fraction:

[tex]\frac{36}{5} =7\frac{1}{5}[/tex]

Find a unit vector u u in R 2 R2 such that u u is perpendicular to v . v. How many such vectors are there

Answers

Answer: hello some part of your question is missing

Let v=〈−2,5〉 in R^2,and let y=〈0,3,−2〉 in R^3.

Find a unit vector u in R^2 such that u is perpendicular to v. How many such vectors are there?

answer:

One(1) unit vector ( < 5/√29,  2 /√29 >  ) perpendicular to 〈−2,5〉

Step-by-step explanation:

let  

u = < x , y > ∈/R^2  be perpendicular to  v = < -2, 5 > ------ ( 1 )

hence :

-2x + 5y = 0

-2x = -5y

x = 5/2 y

back to equation 1

u = < 5/2y, y >

∴ || u || = y/2 √29

u   = < 5 /2 y * 2 / y√29 ,  y*2 / y√29 >

    = < 5/√29,  2 /√29 >  ( unit vector perpendicular to < -2, 5 > )

Hello, Brainly community!

This question is for all of those Calculus people out there.

The volume of a swimming pool is changing with respect to time, such that the volume is given by W(t), where W(t) is measured in cubic centimeters and t is measured in seconds. A tangent line is shown for W(t) at t = 3 seconds. Determine the best estimate for the value of the instantaneous rate of change of W(t) when t = 3.
(I've narrowed down the answer choices to 2, and just really need to find the right way of thinking to find the answer)

(A) W(lim t) as t goes to 3.
(B) [W(3.1) - W(2.9)] / 0.2.

Thank you in advance!

Answers

Answer:

(B)  [tex]\displaystyle \frac{W(3.1) - W(2.9)}{0.2}[/tex]

General Formulas and Concepts:

Calculus

Limits

Derivatives

The definition of a derivative is the slope of the tangent line.

Derivative Notation

Instantaneous Rates

Tangent Line: [tex]\displaystyle f'(x) = \frac{f(b) - f(a)}{b - a}[/tex]

Step-by-step explanation:

Since we are trying to find a rate at which W(t) changes, we must find the derivative at t = 3.

We are given 2 close answer choices that would have the same numerical answer but different meanings:

(A)  [tex]\displaystyle \lim_{t \to 3} W(t)[/tex](B)  [tex]\displaystyle \frac{W(3.1) - W(2.9)}{0.2}[/tex]

If we look at answer choice (A), we see that our units would simply just be volume. It would not have the units of a rate of change. Yes, it may be the closest numerically correct answer, but it does not tell us the rate at which the volume would be changing and it is not a derivative.

If we look at answer choice (B), we see that our units would be cm³/s, and that is most certainly a rate of change. Answer choice (B) is also a derivative at t = 3, and a derivative tells us what rate something is changing.

∴ Answer choice (B) will give us the best estimate for the value of the instantaneous rate of change of W(t) when t = 3.

Topic: AP Calculus AB/BC (Calculus I/I + II)

Unit: Differentiation

Book: College Calculus 10e

A farmer builds a fence to enclose a rectangular pasture. He uses 155 feet of fence. Find the total area of the pasture if it is 45.5 feet long. The length is 6 and the height is 5 what is the width?

Answers

Answer: [tex]1456\ ft^2[/tex]

Step-by-step explanation:

Given

Length of whole fence is 155 feet

If the length of rectangle is 45.5 ft

Suppose width is w

Length of whole fence is perimeter which is given by

[tex]\Rightarrow 155=2(45.5+w)\\\Rightarrow 77.5=45.5+w\\\Rightarrow w=32\ ft[/tex]

Area of the rectangle is given by the product of length and width

[tex]\Rightarrow A=lw\\\Rightarrow A=45.5\times 32\\\Rightarrow A=1456\ ft^2[/tex]

Thus, total area of pasture is [tex]1456\ ft^2[/tex]

HELP SOMEONE PLEASE!!!!!!!!!!!!!!!!!!!

Answers

Answer:

3/10

Step-by-step explanation:

The question asks for the probability of finding 4 or more, so first, you must add up all of the teenagers that have 4 or more shoes. Both the 4 and 5 categories fit, so add 20 and 10. Then, find the total number of people in the sample size. Overall, 100 teenagers are represented. Finally, put the number of people that have 4 or more shoes, 30, over the sample size, 100. This equals 30/100 which should be simplified to 3/10.

A parabola opens upward. The parabola goes through the point (3,-1),
and the vertex is at (2,-2).

Find the value of A for the parabola. Show your work. Use Part 1 and 2 to write the equation of the parabola.

Answers

Answer:

a=1

Step-by-step explanation:

Hopefully this helps :)

The equation of the parabola is: y = (x - 2)² - 2. Finding the value of A

The vertex of the parabola is at (2,-2). Since the parabola opens upward, the equation of the parabola will be of the form:

y = A(x - 2)² - 2

We can plug the point (3,-1) into this equation to find the value of A.

-1 = A(3 - 2)² - 2

Simplifying the right side of the equation, we get:

-1 = A - 2

Adding 2 to both sides of the equation, we get:

1 = A

Therefore, the value of A is 1.

Writing the equation of the parabola

The equation of the parabola is:

y = (x - 2)² - 2

To know more about parabola:

https://brainly.com/question/11911877

#SPJ2

y = RootIndex 3 StartRoot x EndRoot. y = negative (0.4) RootIndex 3 StartRoot x minus 2 EndRoot
Which of the following describes the graph of the transformed function compared with the parent function? Select all that apply.

Answers

Answer:

- Reflected over the x-axis  

- Compressed by a factor of 0.4.

- Translated 2 units left

Step-by-step explanation:

Given

[tex]y = \sqrt[3]{x}[/tex]

[tex]y' = -(0.4)\sqrt[3]{x-2}[/tex]

Required

The transformation from y to y'

First, y is reflected over the x-axis.

The transformation rule is:

[tex](x,y) \to (x,-y)[/tex]

So, we have:

[tex]y = \sqrt[3]{x}[/tex] becomes

[tex]y' = -\sqrt[3]{x}[/tex]

Next, it was compressed by a scale factor of 0.4

The rule is:

[tex]y' = k * y[/tex]

Where k is the scale factor (i.e. k = 0.4)

So, we have:

[tex]y' = 0.4 * -\sqrt[3]{x}[/tex]

[tex]y' = -(0.4)\sqrt[3]{x}[/tex]

Lastly, the function is translated 2 units left;

The rule is:

[tex](x,y) \to (x-2,y)[/tex]

So, we have:

[tex]y' = -(0.4)\sqrt[3]{x - 2}[/tex]

Answers:

-reflected over the x-axis

-translated 2 units right

-compressed by a factor of 0.4

solve in attachment .​

Answers

Answer:

2 ( Option A )

Step-by-step explanation:

The given integral to us is ,

[tex]\longrightarrow \displaystyle \int_0^1 5x \sqrt{x}\ dx [/tex]

Here 5 is a constant so it can come out . So that,

[tex]\longrightarrow \displaystyle I = 5 \int_0^1 x \sqrt{x}\ dx [/tex]

Now we can write √x as ,

[tex]\longrightarrow I = \displaystyle 5 \int_0^1 x . x^{\frac{1}{2}} \ dx [/tex]

Simplify ,

[tex]\longrightarrow I = 5 \displaystyle \int_0^1 x^{\frac{3}{2}}\ dx [/tex]

By Power rule , the integral of x^3/2 wrt x is , 2/5x^5/2 . Therefore ,

[tex]\longrightarrow I = 5 \bigg( \dfrac{2}{5} x^{\frac{5}{2}} \bigg] ^1_0 \bigg) [/tex]

On simplifying we will get ,

[tex]\longrightarrow \underline{\underline{ I = 2 }}[/tex]

Step-by-step explanation:

[tex]thank \: you[/tex]

According to the number line, what is the distance between points A and B?

0 6 units
7 units
O 12 units
O 14 units

Answers

Answer:

14 units

Step-by-step explanation:

A = - 2, B = 12

Therefore,

d(A, B) = 12 - (-2) = 12 + 2 = 14 units

Find the H.C.F. of the following numbers using division method
(a) 24, 40

Answers

Answer:

Ok

Thank you for

your time

The expression 2x and x² have the same value for only two values of x. What are these values?​

Answers

Answer:

0 and 2

Step-by-step explanation:

Which of the following is the parent function of all absolute value functions?
f(x) = 3x
f(x) = |x|
f(x) = 2|x|
f(x)=x²

Answers

f(x)=|x|

a parent function is the basic function before it has undergone transformations.

A cinema is doing a promotion to celebrate their 50th anniversary for 1 week. They give

away a free drink to every 98th customer, a free bag of popcorn to every 112th customer and

a free cinema ticket to every 224th customer. Which lucky customer will be the first to

receive all 3 items?​

Answers

Answer:

1,568 customer

Step-by-step explanation:

Find the lowest common multiple of 98, 112, and 224

98 = 98, 196, 294, 392, 490, 588, 686, 784, 882, 980, 1078, 1176, 1274, 1372, 1470, 1568, 1666

112 = 112, 224, 336, 448, 560, 672, 784, 896, 1008, 1120, 1232, 1344, 1456, 1568, 1680, 1792, 1904

224 = 224, 448, 672, 896, 1120, 1344, 1568, 1792, 2016, 2240

The lowest common multiple of 98, 112, and 224 is 1568

Therefore, the 1,568th customer will be the first to receive all 3 iitem

Sketch the graph of each of the following quadratic functions: (a) f(x) = -2x² + 7x + 4 for -1 ≤ x ≤ 5.
Help me with this ques pleasee,i'll mark u as the brainliest!!​

Answers

Answer:

Please find attached the graph of the function created with MS Excel showing the relevant points required to draw an approximate graph of the function on a graph paper

Step-by-step explanation:

The given quadratic function is f(x) = -2·x² + 7·x + 4

The range of the input (x) values = -1 ≤ x ≤ 5

The coefficient of the quadratic is negative -2, the graph is n shape

The intercept form of the function is given as follows;

-2·x² + 7·x + 4 = -1 × (2·x² - 7·x - 4)

-1 × (2·x² - 7·x - 4) = -1 × (2·x² + x - 8·x - 4)

-1 × (2·x² + x - 8·x - 4) = -1 × (x · (2·x + 1) - 4·(2·x + 1))

∴ -1 × (x · (2·x + 1) - 4·(2·x + 1)) = -1 × (2·x + 1)·(x - 4)

∴ f(x) = -2·x² + 7·x + 4 = -1 × (2·x + 1)·(x - 4)

At the x-intercepts, (2·x + 1) = 0 or (x - 4) = 0, which gives;

x = -1/2 or x = 4

Therefore, the x-intercepts are (-1/2, 0), (4, 0)

The equation in vertex form is given as follows;

f(x) = -2·x² + 7·x + 4 = -2·(x² - 7·x/2 + 2)

By applying completing the squares method, to x² - 7·x/2 - 2, we get;

Where x² - 7·x/2 - 2

x² - 7·x/2 = 2

x² - 7·x/2 + (-7/4)² = 2 + (-7/4)² = 81/15

(x - 7/4)² = 81/16

∴ (x - 7/4)² - 81/16 = 0 = x² - 7·x/2 - 2

∴ x² - 7·x/2 - 2 = (x - 7/4)² - 81/16

-2·(x² - 7·x/2 + 2) = -2·((x - 7/4)² - 81/16) = -2·(x - 7/4)² + 81/8

The vertex = (7/4, 81/8)

When x = 0, we get;

f(0) = -2 × 0² + 7 × 0 + 4 = 4

The y-intercept = (0, 4)

The sketch of the function should pass through the x-intercepts (-1/2, 0), (4, 0), the y-intercept (0, 4), and the y-intercept (0, 4), and the vertex, (7/4, 81/8) on a graph sheet

Please find attached a drawing of the function of the function created with MS Excel

Which number line represents the solution set for the inequality 3(8 – 4x) < 6(x – 5)?

A number line from negative 5 to 5 in increments of 1. An open circle is at 3 and a bold line starts at 3 and is pointing to the left.
A number line from negative 5 to 5 in increments of 1. An open circle is at 3 and a bold line starts at 3 and is pointing to the right.
A number line from negative 5 to 5 in increments of 1. An open circle is at negative 3 and a bold line starts at negative 3 and is pointing to the left.
A number line from negative 5 to 5 in increments of 1. An open circle is at negative 3 and a bold line starts at negative 3 and is pointing to the right.

Answers

Answer:

A number line from negative 5 to 5 in increments of 1. An open circle is at 3 and a bold line starts at 3 and is pointing to the right.

Step-by-step explanation:

Given

[tex]3(8 - 4x) < 6(x - 5)[/tex]

Required

Describe the number line

We have:

[tex]3(8 - 4x) < 6(x - 5)[/tex]

Open brackets

[tex]24 - 12x < 6x - 30[/tex]

Collect like terms

[tex]-12x - 6x < -30 - 24[/tex]

[tex]-18x < -54[/tex]

Divide by -18 (the inequality changes)

[tex]x > 3[/tex]

[tex]>[/tex] means that the arrow on the number line points to the right, and it makes use of an open circle that starts from 3.

Answer:

based on what he said the answer is b

Step-by-step explanation:

find the domain of f(x)=sec(2x)

Answers

Answer:

*Refer the image attached

Step-by-step explanation:

*Refer the image attached

It took Sarah 4 days to write a paper, she wrote 12 pages on day 1, 15 pages on day 2 and 9 pages on day 3. If she wrote 12 pages per day , how many pages did she write on the fourth day ?

Answers

Answer:

Option A

Step-by-step explanation:

Sarah took 4 days to write a paper.

She wrote 12 pages per day, so total number of pages she wrote in 4 days = 12 × 4

= 48 pages

On day 1, she wrote number of pages = 12

On day 2, she wrote number of pages = 15

On day 3, she wrote number of pages = 9

On day 4, she wrote number of pages = P

She wrote total number pages in 4 days = 12 + 15 + 9 + P

                                                                    = 36 + P

Therefore, P + 36 = 48

P = 48 - 36

P = 12

She wrote 12 pages on day 4.

Option A is the answer.

What is -2y + -4y. Simplify the answer.

Answers

Step-by-step explanation:

Explanation is in the attachment

hope it is helpful to you

Answer:

[tex]-2y+\left(-4\right)y[/tex][tex]=-2y-4y[/tex][tex]=-6y[/tex]

[tex]-----------[/tex]

hope it helps...

have a great day!!

Other Questions
The ideas of rousseau, voltaire , and montesquieu most influenced (1)the growing Power of priests in the Roman catholic church(2) improvements in The working conditions of factory workers(3) The rise of industrial capitalism(4) moment for political reformAnswer?? when (S)-3-bromo-2,3-dimethylpentane is treated with sodium chloride with water as the solvent, the products formed are ___________. *Select all that apply The following data are available for product no. CK74, manufactured and sold by Ruby Corporation: Maximum capacity with present facilities 11,000 units Total fixed cost (per period) $ 851,400 Variable cost per unit $ 120.00 Sales price per unit $ 186.00 Required:The number of units of CK74 that Ruby must sell to break- even is:________.a. 12,900.b. 4,577.c. 7,095.d. 6,050. Any grammatical errors in the letter pls: Liebe --------,Ich mchte dir sagen, dass mir fuer deine Situation Leid tut. Es scheint mir nicht richtig, dass Leute jemanden tten wollen, nur weil sie eine andere Religion haben.Ich wollte dir etwas immer fragen. Hast du dir Sorgen gemacht, dass Hans nicht mehr mit dir Freunde sein wollen wird, nachdem er alle diese Dinge ber die Juden gehrt hatte?Ich hoffe dir geht es gut,------ Prove that the expression A + B - C is equivalent to the expression C - B - A if A = 2x - 1, B = 3x + 1 and C = 5x. What was the iron curtain? Why didChurchill choose that term? An airplane flies 105 miles in hour. How far can it fly in 1 hours at the same rate of speed? Suppose you invest equal amounts in a risky asset with an expected return of 16% and a standard deviation of returns of 18% and a risk-free asset with an interest rate of 4%. Calculate the standard deviation of the returns on the resulting portfolio. The U.S. took a more active role in preventing the spread of Communism in Latin America versus Asia or Europe because:it was less expensive to fight Communism in Latin America.the Soviet Union had more troops stationed in Asia and Europe, posing a greater military threat.Latin America was closer to the U.S., so it was considered more essential to the country's security.All of these choices are correct. The new actor on the set was so __________ to every request that the director had not a single argument with him. in a school project you need to provide a blueprint of the schools rectangular playground .the blueprint dimensions of the playground are 23/147 yd x 3/14 yd after reducing them by the factor of 2/147 what are the original dimensions if the playground in yards Suppose a term of a geometric sequence is a4 = 121.5 and the common ratio is 3. Write the formula for this sequence in the form an = a1 rn1. Explain how you arrived at your answer. The Herodians supported Christ's ministry. True False Draw the major product that is obtained when (2S,3S)-2-Bromo-3-phenylbutane is treated with sodium ethoxide. IM BEGGING PLEASE I NEED THE ANSWER TO THIS :C CAN SOMEONE HELP ME AND EXPLAIN ASAP THANKS! What is the inverse of function f? f(x)=10/9+11 find the gradients of line a and b What is the value of x if 2/ 3 - 2 = -4 ? A cost that remains fixed over limited ranges of volumes but changes by a lump sum when volume changes occur outside these limited ranges is called a _____ cost. multiple choice fixed variable mixed step-wise curvilinear